csat introduction

33
Add : D/108, Sec-2, Noida (U.P.), Pin - 20 1301 Email id : [email protected] Call : 09582948810, 09953007628, 0120-2440265 CSA CSA CSA CSA CSAT T T INTR INTR INTR INTR INTRODUCTION ODUCTION ODUCTION ODUCTION ODUCTION

Upload: shriram-iyer

Post on 18-Jul-2016

54 views

Category:

Documents


1 download

DESCRIPTION

intro on csat 2015

TRANSCRIPT

Page 1: CSAT Introduction

Add : D/108, Sec-2, Noida (U.P.), Pin - 20 1301Email id : [email protected]

Call : 09582948810, 09953007628, 0120-2440265

CSACSACSACSACSATTTTT

INTRINTRINTRINTRINTRODUCTIONODUCTIONODUCTIONODUCTIONODUCTION

Page 2: CSAT Introduction

CSAT

CHRO

NICLE

IAS A

CADEM

Y

2

CSAT SYLLABUS

1. COMPREHENSION

2. INTERPERSONAL SKILLS INCLUDING COMMUNICATION SKILLS

3. LOGICAL REASONING AND ANALYTICAL ABILITY

4. DECISION-MAKING AND PROBLEM-SOLVING

5. GENERAL MENTAL ABILITY

6. BASIC NUMERACY (NUMBERS AND THEIR RELATIONS, ORDERS OF MAGNITUDE,ETC.) (CLASS X LEVEL), DATA

7. INTERPRETATION (CHARTS, GRAPHS, TABLES, DATA SUFFICIENCY ETC. - CLASS XLEVEL)

8. ENGLISH LANGUAGE COMPREHENSION SKILLS (CLASS X LEVEL)

CSACSACSACSACSAT INTRODUCTIONT INTRODUCTIONT INTRODUCTIONT INTRODUCTIONT INTRODUCTION

Page 3: CSAT Introduction

3CSAT

CHRO

NICLE

IAS A

CADEM

Y

The new Recruitment Process adopted by theUPSC for Civil Services Prelims Exam is called theCSAT or the Civil Services Aptitude Test which cameinto effect from the Civil Services Examination, 2011.The scheme of this examination has been revised forthe preliminary examination with the objective ofgiving a level playing-field to all candidates. In additionto this, changes have been introduced in the CivilServices (Main) Examination w.e.f. year 2013completing the overall reforms initiated in 2011.

Unlike Paper 1 where quantity of questions is amajor determinant, Paper 2 has 80 questions andcannot be tamed without solving quality questions.Since the preliminary exam in particular is a test ofendurance the candidate needs to prepare himself onthe physical as well as mental level. The time is themain constraint in solving the questions in Paper 2.On an average one gets around 90 seconds for eachquestion. The comprehension section, along withlogical reasoning, is the most time consuming sectionof paper 2. So, one has to judiciously divide one’s timein the examination.

The Civil Services (Preliminary) Examination, 2011was conducted on 12th June, 2011. A record numberof 4,72,290 candidates applied for this examination,out of which 2,43,003 candidates actually appeared.Out of this, 11984 candidates were declared qualifiedfor the Main written examination held in October-November. The Civil Services (Preliminary)Examination, 2012 was conducted on 20th May, 2012.As per the government records, 5,36,506 candidatesapplied for this examination, out of which 2,71,422candidates actually appeared. 13,092 candidates weredeclared qualified for the Main written examinationheld in October, 2012. The statistics suggest thatapproximately 5% of the appeared candidates aredeclared successful in preliminary examination. Thisfact highlights the importance of Paper 2 in clearingthe first stage itself.

On the basis of analysis of 2011 Prelims Paper-2,the break-up for different sections in terms of numberof questions is as follow–

CHRONICLEIAS ACADEMYA CIVIL SERVICES CHRONICLE INITIATIVE

WHY CSAT CAN'T

BE IGNORED

Sections Number of questions asked

2013 2012 2011

Comprehension 23 (9 passages) 32 (8 passages) 27 (7 passages)

English Comprehension 8 (3 passages) 8 (3 passages) 9 (3 passages)

Decision Making and Interpersonal Skill 6 7 8

Quantitative Aptitude & Logical Reasoning 43 33 36

From the above analysis it can be concluded thatthere are some important areas like Comprehension,Decision Making, Quantitative Aptitude andReasoning which constitute the most significant sectionof the question paper.

In terms of Comprehension, Candidates need toimprove their comprehension power asComprehension constitutes half of the paper. Acomprehension exercise consists of a passage uponwhich questions are set to test the candidate’s abilityto understand the content of the given text and inferinformation and meaning from it. For doing well incomprehension one should keep few points in mindwhile attempting this. Logical reasoning and analytical

ability is another important area. It had as many as 13questions in 2011 exam and candidates are requiredto solve problems related to syllogism, sittingarrangement, series completion, puzzle test, logicalsequence etc.

In a way preparing well for Paper 2 is a must forclearing the preliminary examination. One should havea more focused approach while preparing Paper 2. Italso covers limited sections or topics as compared toPaper-1, which covers a galaxy of topics ranging fromhistory to space and science. Statistically also, theweightage of questions in Paper-2 is more than that ofthe questions in Paper-1. ���

Page 4: CSAT Introduction

CSAT

CHRO

NICLE

IAS A

CADEM

Y

4

Comprehension is the classic entry in CSAT syllabusas it is not to test your language skill, but to test yourmoral and ethical aptitude, understanding ofgovernment programmes and policies, social problems,ability to comprehend boring reports, etc.

As an IAS aspirant you are expected to read,assimilate reason, draw inferences and apply yourlearning to different situations. As administrator youwill have to read reports, infer, make strategies andplan. As the time you spend on these documentsimpacts the efficiency and productivity of yourdivision, you must find a way to work speedily andclear the tasks as per requirements and not be thebottleneck where work comes to a standstill.

You wonder how you can enhance yourcompetence and reduce time spent, whether ondeciphering reports, documents, etc. The way toimprove your reading comprehension is evidentlymaking a habit of reading at least a few pages ofeditorials in daily newspapers such as the Hindu, theEconomist or any other daily.

Some Previous Years Questions are as follows:

Read the following passages and answer theitems that follow each passage. Your answers tothese items should be based on the passages only.

Passage - 1

The law in many parts of the world increasinglyrestricts the discharge of agricultural slurry intowatercourses. The simplest and often the mosteconomically sound practice returns the material tothe land as semisolid manure or as sprayed slurry. Thisdilutes its concentration in the environment to whatmight have occurred III a more primitive andsustainable type of agriculture and converts pollutantinto fertilizer. Soil microorganisms decompose theorganic components of sewage and slurry and mostof the mineral nutrients become available to beabsorbed again by the vegetation.

The excess input of nutrients, both nitrogen andphosphorus - based, from agricultural runoff (and

human sewage) has caused many 'healthy'oligotrophic lakes (low nutrient concentrations, lowplant productivity with abundant water weeds, andclear water) to change to eutrophic condition wherehigh nutrient inputs lead to high phytoplanktonproductivity (sometimes dominated by bloom-formingtoxic species). This makes the water turbid, eliminateslarge plants and, in the worst situations, leads toanoxia and fish kills; so called cultural eutrophication.Thus, important ecosystem services arc lost, includingthe provisioning service of wild-caught fish and thecultural services associated with recreation.

The process of cultural eutrophication of lakes hasbeen understood for some time. But only recently didscientists notice huge 'dead zones' in the oceans nearriver outlets, particularly those draining largecatchment areas such as the Mississippi in NorthAmerica and the Yangtze in China. The nutrient-enriched water flows through streams, rivers and lakes,and eventually to the estuary and ocean where theecological impact may be huge, killing virtually allinvertebrates and fish in areas up to 70,000 km2 inextent. More than 150 sea areas worldwide are nowregularly starved of oxygen as a result ofdecomposition of algal blooms, fuelled particularly bynitrogen from agricultural runoff of fertilizers andsewage from large cities. Oceanic dead zones aretypically associated with industrialized nations andusually lie off'- 'countries that subsidize theiragriculture, encouraging farmers to increaseproductivity and use more fertilizer.

1. According to the passage, why should thedischarge of agricultural slurry intowatercourses be restricted?

1. Losing nutrients in this way is not a goodpractice economically.

2. Watercourses do not contain themicroorganisms that can decompose organiccomponents of agricultural slurry.

3. The discharge may lead to theeutrophication of water bodies.

Select the correct answer using the codes givenbelow

CHRONICLEIAS ACADEMYA CIVIL SERVICES CHRONICLE INITIATIVE

PREVIOUS YEARS'

QUESTIONS OF CSAT

COMPREHENSION

Page 5: CSAT Introduction

5CSAT

CHRO

NICLE

IAS A

CADEM

Y

(a) 1 only (b) 2 and 3 only(c) 1 and 3 only (d) 1, 2 and 3

2. The passage refers to the conversion of "pollutantto fertilizer". What is pollutant and what isfertilizer in this context?

(a) Decomposed organic component of slurryis pollutant and microorganisms in soilconstitute fertilizer.

(b) Discharged agricultural slurry is pollutantand decomposed slurry in soil is fertilizer.

(c) Sprayed slurry is pollutant and watercourseis fertilizer.

(d) None of the above expressions is correct inthis context.

3. According to the passage, what are the effectsof indiscriminate use of fertilizers?

1. Addition of pollutants to the soil and water.2. Destruction decomposer of microorganisms

in soil.3. Nutrient enrichment of water bodies.4. Creation of algal blooms.

Select the correct answer from the codes givenbelow:

(a) 1, 2 and 3 only (b) 1, 3 and 4 only(c) 2 arid 4 only (d) 1, 2, 3 and 4

4. What is/are the characteristics of a water bodywith cultural eutrophication?

1. Loss of ecosystem services2. Loss of flora and fauna3. Loss of mineral nutrients

Select the correct answer using the code givenbelow

(a) 1 only (b) 1 and 2 only(c) 2 and 3 only (d) 1, 2 and 3

5. What is the central theme of this passage?

(a) Appropriate legislation is essential to protectthe environment.

(b) Modern agriculture is responsible for thedestruction of environment.

(c) Improper waste disposal from agriculturecan destroy the aquatic ecosystems.

(d) Use of chemical fertilizers is undesirable inagriculture.

Passage - 2

The miseries of the world cannot be cured byphysical help only. Until man's nature changes, hisphysical needs will always arise, and miseries willalways be felt, and no amount of physical help will

remove them completely. The only solution of theproblem is to make mankind pure. Ignorance is themother of evil and of all the misery we sec. Let menhave light, let them be pure and spiritually strong andeducated; then alone will misery cease in the world.We may convert every house in the country into acharitable asylum, we may fill the land with hospitals,but human misery will continue until man's characterchanges.

6. According to the passage, which of thefollowing statements is most likely to be true asthe reason for man's miseries?

(a) The poor economic and social conditionsprevailing in society.

(b) The refusal on the part of man to changehis character.

(c) The absence of physical and material helpfrom his society.

(d) Ever increasing physical' needs due tochanging social structure.

7. With reference to the passage, the followingassumptions have been made:

1. The author gives primary importance tophysical and material help in eradicatinghuman misery.

2. Charitable homes, hospitals, etc. can removehuman misery to a great extent.

Which of the assumptions is/are valid?

(a) 1 only (b) 2 only(c) Both 1 and 2 (d) Neither 1 nor 2

Passage 3

Crude mineral oil comes out of the earth as a thickbrown or black liquid with a strong smell. It is acomplex mixture of many different substances, eachwith its own individual qualities. Most of them arecombinations of hydrogen and carbon in varyingproportions. Such hydrocarbons are also found inother forms such as bitumen, asphalt and natural gas.Mineral oil originates from the carcasses of tinyanimals and from plants that live in the sea. Overmillions of years, these dead creatures form largedeposits under the sea-bed; and ocean currents coverthem with a blanket of sand and silt. As this mineralhardens, it becomes sedimentary rock and effectivelyshuts out the oxygen, so preventing the completedecomposition of the marine deposits underneath. Thelayers of sedimentary rock become thicker and heavier.Their pressure produces heat, which transforms thetiny carcasses into crude oil in a process that is stillgoing on today.

Page 6: CSAT Introduction

CSAT

CHRO

NICLE

IAS A

CADEM

Y

6

8. Mineral oil deposits under the sea do not getcompletely decomposed because they

(a) are constantly washed by the oceancurrents.

(b) become rock and prevent oxygen fromentering them.

(c) contain a mixture 'of hydrogen and carbon.(d) are carcasses of organisms lying in saline

conditions.

9. Sedimentary rock leads to the formation of oildeposits because

(a) there are no saline conditions below it.(b) it allows some dissolved oxygen to enter

the dead organic matter below it.(c) weight of overlying sediment layers causes

the production of heat.(d) it contains the substances that catalyze the

chemical reactions required to change deadorganisms into oil.

Passage 4

Financial markets J n India have acquired greaterdepth and liquidity over the years. Steady reforms since1991 have led to growing linkages and integration ofthe Indian economy and its financial system with theglobal economy. Weak global economic prospects andcontinuing uncertainties the in international financialmarkets therefore, have had their impact on theemerging market economies. Sovereign risk concerns,particularly in the Euro area, affected financial marketsfor the greater part of the year, with the contagion ofGreece's sovereign debt problem spreading to India andother economies by way of higher-than-normal levelsof volatility.

The funding constraints in international financialmarkets could impact both the availability and cost offoreign funding for banks and corporates. Since theIndian financial system is bank dominated, banks'ability to withstand stress is critical to overall financialstability. Indian banks, however, remain robust,notwithstanding a decline in capital to risk-weightedassets ratio and a rise in non-performing asset levelsin the recent past. Capital adequacy levels remainabove the regulatory requirements. The financialmarket infrastructure continues to function withoutany major disruption. With further globalization,consolidation, deregulation, and diversification of thefinancial system, the banking business may becomemore complex and riskier. Issues like risk and liquiditymanagement and enhancing skill therefore assumegreater significance.

10. According to the passage, the financial marketsin the emerging market economies includingIndia had the adverse impact in recent yearsdue to

1. weak global economic prospects.2. uncertainties in the international financial

markets.3. sovereign risk concerns in the Euro area.4. bad monsoons and the resultant crop loss.

Select the correct answer using the code givenbelow:

(a) 1 and 2 only (b) 1, 2 and 3(c) 2 and 3 only (d) 2, 3 and 4

11. The Indian financial markets are affected byglobal changes mainly due to the

(a) increased inflow of remittances fromabroad.

(b) enormous increase in the foreign exchangereserves.

(c) growing global linkages and integration ofthe Indian financial markets.

(d) contagion of Greece's sovereign debtproblem.

12. According to the passage, in the Indian financialsystem, banks' ability to with stand stress iscritical to ensure overall financial stabilitybecause Indian financial system is

(a) controlled by the Government of India.(b) less integrated with banks.(c) controlled by Reserve Bank of India.(d) dominated by banks.

13. Risk and liquidity management assumes moreimportance in the Indian banking system infuture due to

1. further globalization.2. more consolidation and deregulation of the

financial system.3. further diversification of the financial

system.4. more financial inclusion in the economy.

Select the correct answer using the code givenbelow:

(a) 1, 2 and 3 (b) 2, 3 and 4(c) 1 and 2 only (d) 3 and 4 only

Passage - 5

A number of empirical studies find that farmersare risk-averse, though only moderately in many cases.There is also evidence to show that farmers' riskaversion results in cropping patterns and input use

Page 7: CSAT Introduction

7CSAT

CHRO

NICLE

IAS A

CADEM

Y

designed to reduce risk rather than to maximizeincome. Farmers adopt a number of strategies tomanage and cope with agricultural risks. These includepractices like crop and field diversification, non-farmemployment, storage of stocks and strategic migrationof family members. There are also institutions rangingfrom share tenancy to kinship, extended family andinformal credit agencies. One major obstacle to risksharing by farmers is that the same type of risks canaffect a large number of farmers in the region.Empirical studies show that the traditional methodsare not adequate. Hence there is a need for policyinterventions, especially measures that cut acrossgeographical regions.

Policies may aim at tackling agricultural risksdirectly or indirectly. Examples of risk-specific policiesarc crop insurance, price stabilization and thedevelopment of varieties resistant to pests and diseases.Policies which affect risk indirectly are irrigation,subsidized credit and access to information. No singlerisk-specific policy is sufficient to reduce risk and iswithout side-effects, whereas policies not specific torisk influence the general situation and affect risks onlyindirectly. Crop insurance, as a policy measure totackle agricultural risk directly, deserves carefulconsideration in the Indian context and in many otherdeveloping countries because the majority of farmersdepend on rain-fed agriculture and in many areas yieldvariability is the predominant cause of their incomeinstability.

14. The need for policy intervention to mitigate risksin agriculture is because

(a) farmers are extremely risk-averse.(b) farmers do not know how to mitigate risks.(c) the methods adopted by farmers and

existing risk sharing institutions are notadequate.

(d) majority of farmers depend on rain-fedagriculture.

15. Which of the following observations emergesfrom the above passage?

(a) One can identify a single policy that canreduce risk without any side-effect.

(b) No single risk-specific policy is sufficient toreduce agricultural risk.

(c) Policies which affect risk indirectly caneliminate it.

(d) Government's policy intervention canmitigate agricultural risk completely

16. Consider the following statements:

I. A primary group is relatively smaller in size.II. Intimacy is an essential characteristic of a

primary group.

III. A family may be an example of a primarygroup.

In the light of the above statements, which oneof the following is true?

(a) All families are primary groups.(b) All primary groups are families.(c) A group of smaller size is always a primary

group.(d) Members of a primary group know each

other intimately.

Passage - 6

Malnutrition most commonly occurs between theages of six months and two years. This happens despitethe child's food requirements being less than that ofan older child. Malnutrition is often attributed topoverty, but it has been found that even in householdswhere adults eat adequate quantities of food, morethan 50 per cent of children-under-five do not consumeenough food. The child's dependence on someone elseto feed him/her is primarily responsible for themalnutrition. Very often the mother is working andthe responsibility of feeding the young child is left toan older sibling. It is therefore crucial to increaseawareness regarding the child's food needs and howto satisfy them.

17. According to the passage, malnutrition inchildren can be reduced

(a) if the children have regular intake of food.(b) after they cross the age of five.(c) if the food needs of younger children are

known.(d) if the responsibility of feeding younger

children is given to adults.

18. According to the author, poverty is not the maincause of malnutrition, but the fact that

1. taking care of younger ones is not a priorityfor working mothers.

2. awareness of nutritional needs is notpropagated by the Public Health authorities.

Select the correct answer using the code givenbelow:

(a) 1 only (b) 2 only(c) Both 1 and 2 (d) Neither 1 nor 2

Passage -7

The subject of democracy has become severelymuddled because of the way the rhetoric surroundingit has been used in recent years, There IS, increasingly,an oddly confused dichotomy between those who

Page 8: CSAT Introduction

CSAT

CHRO

NICLE

IAS A

CADEM

Y

8

want to 'impose' democracy on countries In the non-Western world (in these countries' 'own interest', ofcourse) and those who are opposed to such 'imposition'(because of the respect for the countries' 'own ways').But the entire language of 'imposition', used by bothsides, is extraordinarily inappropriate since it makesthe implicit assumption that democracy belongsexclusively to the West, taking it to be a quintessentially'Western' idea which has originated and flourishedonly in the West.

But the thesis and the pessimism it generates aboutthe possibility of democratic practice in the worldwould be extremely hard to justify. There were severalexperiments in local democracy in ancient India.Indeed, in understanding the roots of democracy inthe world, we have to take an interest in the history ofpeople participation and public reasoning in differentparts of the world. We have to look beyond thinkingof democracy only in terms of European and Americanevolution. We would fail to understand the pervasivedemands for participatory living, on which Aristotlespoke with far-reaching insight, if we take democracyto be a kind of a specialized cultural product of theWest.

It cannot, of course, be doubted that theinstitutional structure of the contemporary practice ofdemocracy is largely the product of European andAmerican experience over the last few centuries. Thisis extremely important to recognize since thesedevelopments in institutional formats were immenselyinnovative and ultimately effective. There can be littledoubt that there is a major 'Western' achievement here.

19. Which of the following is closest to the view ofdemocracy as mentioned in the above passage?

(a) The subject of democracy is a muddle dueto a desire to portray it as a Westernconcept, 'alien' to non-Western countries.

(b) The language or imposition of democracyis inappropriate. There is, however, a needto consider this concept in the backdrop ofculture of 'own ways' of non-Westernsociety.

(c) While democracy is not essentially aWestern idea belonging exclusively to theWest, the institutional structure of currentdemocratic practices has been theircontribution.

(d) None of the statements (a), (b) and (c) givenabove is correct.

20. With reference to the passage, the followingassumptions have been made:

1. Many of the non-Western countries areunable to have democracy because theytake democracy to be a specialized culturalproduct of the West.

2. Western countries are always trying toimpose democracy on non-Westerncountries.

Which of the above is/are valid assumption/assumptions?

(a) 1 only (b) 2 only(c) Both 1 and 2 (d) Neither 1 nor 2

Passage - 8

Corporate governance is based on principles suchas conducting the business with all integrity andfairness, being transparent with regard to alltransactions, making all the necessary disclosures anddecisions, complying with all the laws of the land,accountability and responsibility towards thestakeholders and commitment to conducting businessin an ethical manner. Another point which ishighlighted on corporate governance is the need forthose in control to be able to distinguish between whatare personal and corporate funds while managing acompany.

Fundamentally, there is a level of confidence thatis associated with a company that is known to havegood corporate governance. The presence of an activegroup of independent directors on the boardcontributes a great deal towards ensuring confidencem the market. Corporate governance is known to beone of the criteria that foreign institutional investorsare increasingly depending on when deciding on whichcompanies to invest in. It is also known to have apositive influence on the share price of the company.Having a clean image on the corporate governancefront could also make it easier for companies to sourcecapital at more reasonable costs. Unfortunately,corporate governance often becomes the centre ofdiscussion only after the exposure of a large scam.

21. According to the passage, which of thefollowing should be the practice/practices ingood corporate governance?

1. Companies should always comply withlabour and tax laws of the land.

2. Every company in the country should havea government 'representative as one of theindependent directors on the board toensure transparency.

3. The manager of a company should neverinvest his personal, funds in the company.

Page 9: CSAT Introduction

9CSAT

CHRO

NICLE

IAS A

CADEM

Y

Select the correct answer using the codes givenbelow:

(a) 1 only (b) 2 and 3 only(c) 1 and 3 only (d) 1, 2 and 3

22. According to the passage, which of thefollowing is/are the major benefit/benefits ofgood corporate governance?

1. Good corporate governance leads toincrease in share price of the company.

2. A company with good corporategovernance always increases its businessturnover rapidly.

3. Good corporate governance is the maincriterion for foreign institutional investorswhen they decide to buy a company.

Select the correct answer using the codes givenbelow:

(a) 1 only (b) 2 and 3 only(c) 1 and 3 only (d) 1, 2 and 3

Passage 9

Ecological -research over the last quarter of thecentury has established the deleterious effects ofhabitat fragmentation due to mining, highways andsuch other intrusions on forests. When a large block offorests gets fragmented into smaller bits, the edges ofall these bits come into contact with human activitiesresulting in the degradation of the entire forests.Continuity of forested landscapes and corridors getsdisrupted 'affecting several extinction-prone species ofwildlife. Habitat fragmentation, is therefore consideredas the most serious threat to biodiversity conservation.Ad hoc grants of forest lands to mining companiescoupled with rampant, illegal mining is aggravatingthis threat.

23. What is the central focus of this passage?

(a) Illegal mining in forests(b) Extinction of wildlife(c) Conservation of nature(d) Disruption of habitat

24. What is the purpose of maintaining thecontinuity of forested landscapes and corridors?

1. Preservation of biodiversity.2. Management of mineral resources.3. Grant of forest lands for human activities.

Select the correct answer using the codes givenbelow:

(a) 1 only (b) 1 and 2(c) 2 and 3 (d) 1, 2 and 3

Passage – 10

The poor, especially in market economies, need thestrength that collectivizes offer for creating moreeconomic, social and political space for themselves, forenhancing their socioeconomic well-being and voice,and as a protection against free market individualism.It has been argued that a group approach to farming,especially in the form of bottom up agriculturalproduction collectivities, offers substantial scope forpoverty alleviation and empowering the poor as wellas enhancing agricultural productivity. To realize thispotential, however, the groups would need to bevoluntary in nature, small in size, participative indecision making and equitable in work sharing andbenefit distribution. There are many notable examplesof such collectivities to be found in varied contexts,such as in the transition economies. All of them bearwitness to the possibility of successful cooperationunder given conditions. And although the genderimpact of the family cooperatives in the transitioneconomies are uncertain, the Indian examples ofwomen-only groups farming offer considerablepotential for benefiting women.

25. Agricultural collectivities such as group basedfarming can provide the rural poor

1. Empowerment.2. Increased agricultural productivity.3. Safeguard against exploitative markets.4. Surplus production of agricultural

commodities.

Select the correct answer using the codes givenbelow:

(a) 1, 2, 3, and 4 (b) 1, 2 and 3 only(c) 2 and 4 only (d) 1, 3 and 4 only

26. What does the author imply by “genderimpact”?

(a) Women are doubtful participants incooperatives.

(b) Family cooperatives may not includewomen.

(c) Women benefiting from group farming.(d) Women’s role in transition economies is

highly restrictive.

27. Consider the following assumptions:

1. It is imperative for transition economies tohave agricultural collectivities.

2. Agricultural productivity can be increasedby group approach to farming.

With reference to the above passage, which ofthese assumptions is/are valid?

Page 10: CSAT Introduction

CSAT

CHRO

NICLE

IAS A

CADEM

Y

10

(a) 1 only (b) 2 only(c) Both 1 and 2 (d) Neither 1 nor 2

Passage – 11

In a typical Western liberal context, deepening ofdemocracy invariably leads to consolidation of ‘liberalvalues”. In the Indian context, democratization istranslated into greater involvement of people not as‘individuals’ which is a staple to liberal discourse, butas communities or groups. Individuals are gettinginvolved in the public sphere not as ‘atomized’individuals but as members of primordial communitiesdrawn on religious or caste identity. Community-identity seems to be the governing force. It is nottherefore surprising that the so-called peripheralgroups continue to maintain their identities withreference to the social groups (caste, religion or sect)to which they belong while getting involved in thepolitical processes despite the fact that their politicalgoals remain more or less identical. By helping toarticulate the political voice of the marginalized,democracy in India has led to ‘a loosening of socialstrictures” and empowered the peripherals to beconfident of their ability to improve the socio-economicconditions in which they are placed. This is a significantpolitical process that had led to a silent revolutionthrough a meaningful transfer of power from theupper caste elites to various subaltern groups withinthe democratic framework of public governance.

28. According to the passage, what does“deepening of democracy” mean in the Westerncontext?

(a) Consolidation of group and class identities.(b) Democratization translated as greater

involvement of people.(c) Democratization as greater involvement of

‘atomized” individuals in the public sphere.(d) None of the statements (a), (b) and (c) given

above is correct in this context.

29. Greater democratization in India has notnecessarily led to

(a) The dilution of caste and communalidentities in the public sphere.

(b) Irrelevance of community identity as agoverning force in Indian politics.

(c) Marginalization of elite groups in society.(d) Relative unimportance of hereditary

identities over class identities.

30. What is the “silent revolution” that has occurredin the Indian democratic process?

(a) Irrelevance of caste and class hierarchies inpolitical processes.

(b) Loosening of social strictures in votingbehavior and patterns.

(c) Social change through transfer of powerfrom upper caste elites to subaltern groups.

(d) All the statements (a), (b) and (c) givenabove are correct in this context.

Passage – 12

Education, without a doubt, has an importantfunctional, instrumental and utilitarian dimension.This is revealed when one asks questions such as 'whatis the purpose of education?' The answer, too often,are to acquire qualifications for employment/upwardmobility', 'wider/higher (in terms of income)opportunities', and 'to meet the needs for trainedhuman power in diverse fields for national humanpower in diverse fields for national development'. Butin its deepest sense education is not instrumentalist.That is to say, it is not to be justified outside of itselfbecause it leads to the acquisition of formal skills or ofcertain desired psychological – social attributes. It mustbe respected in itself. Education is thus not acommodity to be acquired or possessed and then used,but a process of inestimable importance to individualsand society, although it can and does have enormoususe value. Education, then, is a process of expansionand conversion, not in the sense of converting orturning students into doctors or engineers, but thewidening and turning out of the mind the creation,sustenance and development of self-critical awarenessand independence of through. It is an inner process ofmoral – intellectual development.

31. What do you understand by the 'instrumentalist'view of education?

(a) Education is functional and utilitarian is itspurposes.

(b) Education is meant to fulfill human needs(c) The purpose of education is to train the

human intellect.(d) Education is meant to achieve moral

development.

32. According to the passage, education must berespected in itself because

(a) It helps to acquire qualification foremployment

(b) it helps in upward mobility and acquiringsocial status.

(c) it is an inner process of moral andintellectual development

(d) All the (a), (b) and (c) given above arecorrect in this context.

Page 11: CSAT Introduction

11CSAT

CHRO

NICLE

IAS A

CADEM

Y

33. Education is a process in which

(a) students are converted into trainedprofessionals.

(b) opportunities for higher income aregenerated

(c) individuals develop self-critical awarenessand independence of thought.

(d) qualifications for upward mobility areacquired.

Passage – 13

Chemical pesticides lose their role in sustainableagriculture if the pests evolve resistance. The evolutionof pesticide resistance is simply natural selection inaction. It is almost certain to occur when vast numbersof a genetically variable population are killed. One ora few individuals may be unusually resistant (perhapsbecause they possess an enzyme that can detoxify thepesticide). If the pesticide is applied repeatedly, eachsuccessive generation of the pest will contain a largerproportion of resistant individuals. Pest typically havea high intrinsic rate of reproduction and so a fewindividuals in one generation may give rise to hundredsor thousands in the next, and resistance spreads veryrapidly in a population.

This problem was often ignored in the past, eventhough the first case of DDT (dichlorodiphenyltrichloroethane) resistance was reported as early as1946. There is exponential increase in the number ofinvertebrates that have evolved resistance and in thenumber of pesticides against which resistance hasevolved. Resistance has been recorded in every familyof arthropod (including dipterans such as mosquitoesand house flies, as well as beetles, moths, wasps, fleas,lice and mites) as well as in weeds and plant pathogens.Take the Alabama leafworm, a moth pest of cotton, asan example. It has developed resistance in one or moreregions of the world to aldrin, DDT, dieldrin, endrin,lindane and toxaphene.

If chemical pesticides brought nothing butproblems, - if their use was intrinsically and acutelyunsustainable – then they would already have fallenout of widespread use. This has not happened. Instead,their rate of production has increased rapidly. The ratioof cost to benefit for individual agricultural producerhas remained in favour of pesticide use. In the USA,insecticides have been estimated to benefit theagriculture products to benefit the agriculturalproducts to the tune of around $5 for every $spent.

Moreover, in many poorer countries, the prospectof imminent mass starvation, or of an epidemic disease,are so frightening that the social and health costs ofusing pesticides have to be ignored. In general the use

of pesticides is justified by objective measures such as'lives saved', 'economic efficiency of food production'and 'total food produced'. In these very fundamentalsenses, their use may be described as sustainable. Inpractice, sustainability depends on continuallydeveloping new pesticides that keep at last one stepahead of the pests – pesticides that are less persistent,biodegradable and more accurately targeted at thepests.

34. “The evolution of pesticide resistance is naturalselection in action.” What does it actually imply?

(a) It is very natural for many organisms tohave pesticide resistance.

(b) Pesticide resistance among organisms is auniversal phenomenon.

(c) Some individuals in any given populationshow resistance after the application ofpesticides.

(d) None of the statements (a) (b) and (c) givenis correct.

35. With reference to the passage, consider thefollowing statement:

1. Use of chemical pesticides has becomeimperative in all the poor countries of theworld.

2. Chemical pesticides should not have anyrole in sustainable agriculture.

3. One pest can develop resistance to manypesticides.

Which of the statement given above is/arecorrect?

(a) 1 and 2 only (b) 3 only(c) 1 and 3 only (d) 1, 2 and 3

36. Though the problem associated with the use ofchemical pesticides is known for a long time,their widespread use has not waned. Why?

(a) Alternatives to chemical pesticides do notexist at all

(b) New pesticides are not invented at all(c) Pesticides are biodegradable(d) None of the statements (a) (b) and (c) given

is correct.

37. How do pesticides act as agents for the selectionof resistant individuals in any pest population?

1. It is possible that in a pest, population theindividuals will behave differently due totheir genetic makeup

2. Pests do possess the ability to detoxify thepesticides.

Page 12: CSAT Introduction

CSAT

CHRO

NICLE

IAS A

CADEM

Y

12

3. Evolution of pesticide resistance is equallydistributed in pest population.

Which of the statements given above is/arecorrect?

(a) 1 only (b) 1 and 2 only(c) 3 only (d) 1, 2 and 3

38. Why is the use of chemical pesticides generallyjustified by giving the examples of poor anddeveloping countries?

1. Developed countries can afford to do awaywith use of pesticides by adapting toorganic farming, but it is imperative forpoor and developing countries to usechemical pesticides.

2. In poor and developing countries, thepesticide addresses the problem of epidemicdiseases of crops and eases the foodproblem.

3. The social and health costs of pesticide useare generally ignored in poor anddeveloping countries

Which of the statements given above is/arecorrect?

(a) 1 only (b) 1 and 2 only(c) 2 only (d) 1, 2 and 3

39. What does the passage imply?

(a) Alternative options to chemical pesticidesshould be promoted.

(b) Too much use of chemicals is not good forthe ecosystem.

(c) There is no scope for the improvement ofpesticides and making their use sustainable.

(d) Both the statements (a) and (b) above arecorrect.

Passage – 14

Today's developing economies use much lessenergy per capita than developed countries such asthe United States did at similar incomes, showing thepotential for lower-carbon growth. Adaptation andmitigation need to be integrated into a climate-smartdevelopment strategy that increases resilience reducesthe threat of further global warming and improvesdevelopment outcomes. Adaptation and mitigationmeasures can advance development, and prosperitycan raise income and foster better institutions. Ahealthier population living in better-built houses andwith access to bank loans and social securing is betterequipped to deal with a changing climate and itsconsequence. Advancing robust, resilient developmentpolicies that promote adaptation is needed today

because changes in the climate, already begun, willincrease even in the short term.

The spread of economic prosperity has always beenintertwined with adaptation to changing ecologicalconditions. But as growth has altered the environmentand as environmental change has accelerated,sustaining growth and adaptability demands greatercapacity to understand our environment, generate newadaptive technologies and practices, and diffuse themwidely. As economic historians have explained, muchof humankind's creative potential has been directedat adapting to the changing world. But adaptioncannot cope with all the impacts related to climatechange, especially as larger changes unfold in the longterm.

Countries cannot grow out of harm's way fastenough to match the changing climate. And somegrowth strategies, whether driven by the governmentor the market, can also add to vulnerability –particularly if they overexploit natural resources.Under the soviet development plan, irrigated cottoncultivation expanded in water-stressed Central Asiaand led to the near disappearance of the Aral Sea,threatening the livelihoods of fishermen, herders andfarmers. And clearing mangroves the natural coastalbuffers against storm surges – to make way for intensivefarming or housing development, increases thephysical vulnerability of coastal settlements, whetherin Guinea or in Louisiana.

40. Which of the following conditions of growthcan add to vulnerability?

1. When the growth occurs due to excessiveexploitation of mineral resources andforests.

2. When the growth brings about a change inhumankind's creative potential.

3. When the growth is envisaged only forproviding houses and social security to thepeople.

4. When the growth occurs due to emphasison farming only.

Select the correct answer using the codes givenbelow:

(a) 1 only (b) 2, 3 and 4 only(c) 1 and 4 only (d) 1, 2, 3 and 4

41. What does low-carbon growth imply in thepresent context?

1. More emphasis on the use of renewablesources of energy.

2. Less emphasis on manufacturing sector andmore emphasis on agriculture sector.

Page 13: CSAT Introduction

13CSAT

CHRO

NICLE

IAS A

CADEM

Y

3. Switching over from monoculture practicesto mixed farming.

4. Less demand for goods and services.

Select the correct answer using the codes givenbelow:

(a) 1 only(b) 2, 3 and 4 only(c) 1 and 4 only(d) None of the above implies low-carbon

growth

42. Which of the following conditions is/arenecessary for sustainable economic growth?

1. Spreading of economic prosperity more2. Popularising/spreading of adaptive

technologies widely.3. Investing on research in adaptation and

mitigation technology.

Select the correct answer using the codes givenbelow:

(a) 1 only (b) 2 and 3 only(c) 1 and 3 only (d) 1, 2 and 3

43. Which of the following inferences can be madefrom the passage?

1. Rained crops should not be cultivated inirrigated areas.

2. Farming under water-deficient areas shouldnot be a part of development strategy.

Select the correct answer using the codes givenbelow:

(a) 1 only (b) 2 only(c) Both 1 and 2 (d) Nether 1 nor 2

44. Consider the following assumption:

1. Sustainable economic growth demands theuse of creative potential of men.

2. Intensive agriculture can lead to ecologicalbacklash.

3. Spread of economic prosperity can adverselyaffect the ecology and environment.

With reference to the passage, which of theabove assumptions is/are valid?

(a) 1 only (b) 2 and 3 only(c) 1 and 3 only (d) 1 , 2 and 3

45. Which one of the following statementsconstitutes the central theme of this passage?

(a) Countries with greater economic prosperityare better equipped to deal with theconsequences of climate change.

(b) Adaptation and mitigation should beintegrated with development strategies.

(c) Rapid economic growth should not bepursued by both developed and developingcountries.

(d) Some countries resort to overexploitationof natural resources for the sake of rapiddevelopment

Passage – 15

Invasions of exotic species into new geographicareas sometimes occur naturally and without humanagency. However, human action have increased thistrickle to a flood. Human-caused introduction mayoccur either accidentally as a consequence of humantransport, or internationally but illegally to serve someprivate purpose or legitimately to procure some hoped-for public benefit by bringing a pest under control,producing new agricultural products or providingnovel recreational opportunities. Many introducedspecies are assimilated into communities without muchobvious effect. However, some have been responsiblefor dramatic changes to native species and naturalcommunities. For example, the accidental introductionof the brown tree snake Boiga irregularies into Guam,an island in the Pacific, has through nest predationreduced 10 endemic forest bird species to the point ofextinction.

One of the major reasons for the world's greatbiodiversity is the occurrence of centers of endemismso that similar habitats in different parts of the worldare occupied by different groups of species that happento have evolved there. If every species naturally hadaccess to everywhere on the globe, we might expect arelatively small number of successful species to becomedominant in each biome. The extent to which thishomogenization can happen naturally is restricted bythe limited powers of dispersal of most species in theface of the physical barriers that exist to dispersal. Byvirture of the transport opportunities offered byhumans, these barriers have been breached by an ever-increasing number of exotic species. The effects ofintroduction have been to convert a hugely diverserange of local community compositions into somethingmuch more homogeneous.

It would be wrong, however, to conclude thatintroducing species to a region will inevitably cause adecline in species richness there. For example, thereare numerous species of plants, invertebrates andvertebrates found in continental Europe but absentfrom the British Isles (many because they have so farfailed to decolonize after the last glaciations). Theirintroduction would be likely to augment Britishbiodiversity. The significant detrimental effect noted

Page 14: CSAT Introduction

CSAT

CHRO

NICLE

IAS A

CADEM

Y

14

above arise where aggressive species provide a novelchallenge to endemic biotas ill-equipped to deal withthem.

46. With reference to the passage, which of thefollowing statements is correct?

(a) Introduction of exotic species into newgeographical areas always leads to reducedbiodiversity.

(b) Exotic species introduced by man into newareas have always greatly altered the nativeecosystems.

(c) Man is the only reason to convert a hugelydiverse range of local communitycompositions into more homogeneousones..

(d) None of the statements (a), (b) and (c) iscorrect in this context.

47. Why does man introduce exotic species intonew geographical areas?

1. To breed exotic species with local varieties.2. To increase agricultural productivity.3. For beautification and landscaping.

Which of the above statements is/are correct?

(a) 1 only (b) 2 and 3 only(c) 1 and 3 only (d) 1, 2 and 3

48. How is homogenization prevented undernatural conditions?

(a) Evolution of groups of species specific tolocal habitats.

(b) Presence of oceans and mountain ranges.(c) Strong adaptation of groups of species to

local physical and climatic conditions.(d) All the statements (a), (b) and (c) given

above are correct in this context.

49. How have the human beings influenced thebiodiversity?

1. By smuggling live organisms.2. By building highways.3. By making ecosystems sensitive so that new

species are not allowed.4. By ensuring that new species do not have

major impact on local species.

Which of the statements given above are correct?

(a) 1 and 2(b) 2 and 3(c) 1 and 3(d) 2 and 4

50. What can be the impact of invasion of exoticspecies on an ecosystem?

1. Erosion of endemic species.2. Change in the species composition of the

community of the ecosystem.

Select the correct answer using the codes givenbelow:

(a) 1 only (b) 2 only(c) Both 1 and 2 (d) Neither 1 nor 2

���

SOURCES OF PASSAGES

We have also identified the various sourcesemployed by the UPSC to frame paragraphs for thequestions of comprehension. These have beenmentioned as follows:

A. SOCIAL

1. The Concept of a “Creative Society”[CSAT-2011]

[Book: “Readings in Indian Government andPolitics: Class, Caste, Gender (Volume of Essays)”edited by Manoranjan Mohanty; Publisher: SAGE;‘Chapter-15: Conclusion—Social Movements in aCreative Society’ by Manoranjan Mohanty (TheCreative Society: Page-390)]

B. POLITICAL

2. Deepening of Democracy in India:[CSAT-2012]

Transfer of Power from Upper Caste Elites to theMarginalised Subaltern Groups [CSAT-2012]

[Book: “Indian Politics and Society SinceIndependence: Events, Processes and Ideology” byBidyut Chakrabarty; Publisher: Routledge; Conclusion:“Growing Democratization” (Pages-171-172)]

3. Democracy and Development—Do they GoTogether? [CSAT-2012]

[Book: “The Idea of Justice” by Amartya Sen;

Page 15: CSAT Introduction

15CSAT

CHRO

NICLE

IAS A

CADEM

Y

Publisher: Penguin; Chapter: ‘Democracy andDevelopment’]

4. Democracy Not Exclusive Western Concept, ButDevelopment of Institutional Structure

[CSAT-2013] of Democracy—A Western Achievement

[Book: “The Idea of Justice” by Amartya Sen;Publisher: Penguin; Part Four: “Public Reasoning andDemocracy”; Chapter-15: ‘Democracy as PublicReason’]

C. ECONOMIC

5. Indian Economy—A Great Paradox[CSAT-SAMPLE]

[Book: “Contemporary India: Economy, Society,Politics”; Publisher: PEARSON; Part-I, Chapter-2: TheEvolution of Development Strategy SinceIndependence by Ambuja Kumar Tripathy]

6. An Enabling State for Inclusive Growth—TheNeed of the Hour [CSAT-2011]

[The Economic Survey of India 2009-10, Chapter-2: Micro-foundations of Inclusive Growth, Paragraph2.5]

7. The Potential of Farm Collectivities in TransitionEconomies [CSAT-2012]

[Working Paper No. 305 (2010) by Bina Agarwal:“Rethinking Agricultural Production Collectivities”,Institute of Economic Growth, Delhi University;Chapter-7: In Conclusion (Page-31)]

8. Competition Law to Facilitate a Level-PlayingField Among Foreign and Domestic Firms

[CSAT-2012]

[Article in The Economic Times dated 04 Nov 2009:“Competition Law and Inclusive Growth” by MadhavMehra]

9. Climate-Smart Development Strategies ofAdaptation and Mitigation—The Need of theHour [CSAT-2012]

[The World Bank Report: “World DevelopmentReport—Development and Climate Change” (2010);Chapter-1 (Pages-44-45)—‘Growth: Changing CarbonFootprints and Vulnerabilities’]

10. Indian Financial Markets Integrated WithInternational Financial Markets

[CSAT-2013]

[The Economic Survey of India 2011-12, Chapter-5:Financial Intermediation and Markets (Introduction)]

11. Corporate Governance [CSAT-2013]

[Article in The Economic Times dated 18 January2009: “Corporate Governance” by Lisa MaryThomson]

D. SCIENTIFIC/ECOLOGY/ENVIRONMENT

12. Effects of Modifications to Earth’s Ecosystem,[CSAT-2011]

Necessitated by a Rapidly Expanding HumanPopulation

[Textbook: “Life-The Science of Biology” (8thEdition) co-published by Sinauer Associates Inc. andWH Freeman; Chapter-56: “Ecosystems and the GlobalEcology”—56.4 ‘What Services do Ecosystems Provide’(Page-1221)]

13. The Influence of Keystone Species on SpeciesRichness in an Ecosystem [CSAT-2011]

[Textbook: “Life-The Science of Biology” (8thEdition) co-published by Sinauer Associates Inc. andWH Freeman; Chapter-55: “Communities andEcosystems”— (Pages-1062-63) ‘Individual SpeciesMay Influence Community Processes’]

14. Introduction of Exotic Species in NewGeographic Areas and Its Impact onBiodiversity [CSAT-2012]

[Textbook: “Essentials of Ecology” by Colin RTownsend, Michael Begon, John L Harper; Publisher:John Wiley & Sons; Article 14.2.2 Habitat Disruption(Page-462)]

15. Use of Chemical Pesticides in Agriculture—Isit Sustainable? [CSAT-2012]

[Website: www.trinitygreenconsultancy.com;Category: “Species Richness”; Article: ‘Evolution ofResistance to Pesticides” (posted 01 Nov 2012)]

Page 16: CSAT Introduction

CSAT

CHRO

NICLE

IAS A

CADEM

Y

16

16. Habitat Fragmentation—The Most SeriousThreat to Biodiversity Conservation

[CSAT-2013]

[Article in Deccan Herald dated 17 August 2008:“Mining has spelt ecological disaster” by PraveenBhargav]

17. Excessive & Indiscriminate Use of ChemicalFertilizers in Agriculture—

[CSAT-2013]

A Serious Threat to Aquatic Ecosystems

[Textbook: “Essentials of Ecology” by Colin RTownsend, Michael Begon, John L Harper; Publisher:John Wiley & Sons; Chapter 13: Habitat Degradation;Article 13.2.2 Intensive Cropping (Pages-429 & 430)]

E. EDUCATION

18. Will the Right to Education Remain on Paper orBecome a Reality, [CSAT-2011]

Particularly so for Girls?

[Article in The Hindu dated 02 April 2010: “India’s

Children Have a Precarious Right” by Krishna Kumar.]

F. ETHICS & PHILOSOPHY

19. A Moral Act is an Act of Will [CSAT-2011]

[Book: “Ethical Religion” by MohandasKaramchand Gandhi; Chapter-3: “What is MoralAction”]

20. India’s Spiritual Greatness—Myth or Reality?[CSAT-2011]

[Book: “Essays on Ancient India” by Raj Kumar;Publisher: Discovery Publishing House]

21. Ever Increasing Man’s Materialistic NeedsLeads to Endless Miseries; [CSAT-2013]

Only Solution to Change Man’s Character—MakeHim Pure, Educated and Spiritually Strong

[Book: “The Indispensable Vivekananda: AnAnthology for Our Times” by Swami Vivekananda;Editor: Amiya P Sen; Publisher: Permanent Black;Chapter III. b. “Karma Yoga: Service Before Self”(Page-203)]

ENGLISH LANGUAGE

COMPREHENSION

Here the candidates knowledge of English languagerelated to Grammar & Vocabulary will be tested. Thepassages under this category don't have Hindi Versionas these are to test the comprehension of EnglishLanguage.

Comprehension & English Language Comprehensionare two different things. Comprehension will checkcandidates assimilation power of contents whereasEnglish Language Comprehension will check the basicknowledge of English of candidate in terms ofvocabulary & grammar.

Some Previous Years Questions are as follows:

English Passage - 1

Seven-year-old Jim came home from the parkwithout his new bicycle. "An old man and a little boy

borrowed it," he explained. "They are going to bring itback at four o'clock." His parents were upset that hehad given his expensive new bicycle, but were secretlyproud of his kindness and faith. Came four o'clock, nobicycle. The parents were anxious. But at 4:30, the doorbell rang, and there stood a happy man and a boy,with the bicycle and a box of chocolates. Jim suddenlydisappeared into his bedroom, and then came runningout. "All right," he said, after examining the bicycle.''You can have your watch back!"

1. When Jim came home without his bicycle, hisparents

(a) were angry with him.(b) were worried.(c) did not feel concerned.(d) were eager to meet the old man and the

little boy.

���

Page 17: CSAT Introduction

17CSAT

CHRO

NICLE

IAS A

CADEM

Y

2. Jim returned the watch to the old man and thelittle boy because

(a) they gave him chocolates.(b) his father was proud of him.(c) he was satisfied with the condition of his

bicycle.(d) they were late only by 30 minutes.

English Passage - 2

It was already late when we set out for the nexttown, which according to the map was about fifteenkilometres away on the other side of the hills. Therewe felt that we would find a bed for the night.Darkness fell soon after we left the village, but luckilywe met no one as we drove swiftly along the narrowwinding road that led to the hills. As we climbedhigher, it became colder and rain began to fall, makingit difficult at times to see the road. I asked John, mycompanion, to drive more slowly. After we hadtravelled for about twenty kilometres, there was stillno sign of the town which was marked on the map.We were beginning to get worried. Then withoutwarning, the car stopped and we found we had runout of petrol.

3. The author asked John to drive more slowlybecause

(a) the road led to the hills.(b) John was an inexperienced driver.(c) the road was not clearly visible.(d) they were in wilderness.

4. The travellers set out for the town although itwas getting dark because

(a) they were in a hurry.(b) the next town was a short distance away

and was a hill-resort.(c) they were in wilderness.(d) the next town was a short distance away

and promised a good rest for the night.

5. The travellers were worried after twenty kilometres because

(a) it was a lonely countryside.(b) they probably feared of having lost their

way.(c) the rain began to fall(d) it was getting colder as they drove.

English Passage - 3

A stout old lady was walking with her basket downthe middle of a street in Petrograd to the great confusion

of the traffic and no small peril to herself. It was pointedout to her that the pavement was the place for foot-passengers, but she replied, "I m going to walk whereI like. We've got liberty now." It did not occur to thedear lady that if liberty entitled the foot-passenger towalk down the middle of the road it also entitled thetaxi-driver to drive on the pavement, and that the endof such liberty would be universal chaos. Everythingwould be getting in everybody else's way and nobodywould get anywhere. Individual liberty would havebecome social anarchy.

6. It was pointed out to the lady that she shouldwalk on the pavement because she was

(a) a pedestrian. (b) carrying a basket.(c) stout. (d) an old lady.

7. The lady refused to move from the middle ofthe street because

(a) she was not afraid of being killed.(b) she felt that she is entitled to do whatever

she liked.(c) she did not like walking on the pavement.(d) she was confused.

8. The old lady failed to realise that

(a) she was not really free.(b) her liberty was not unlimited.(c) she was an old person.(d) roads are made for motor vehicles only.

English Passage – 4

For fourteen and a half months I lived in my littlecell or room in the Dehradun jail, and I b egan to feelas if I was almost a part of It. I was familiar with everybit of it, I knew every mark and dent on thewhitewashed walls and on the uneven floor and theceiling with its moth-eaten rafters. Ini the little yardoutside I greeted little tufts of grass and odd bits ofstone as old frients. I was not alone in my cell, forseveral colonies of wasp and hornets lived there, andmany lizards found a home behind the rafters,emerging in the evenings in search of prey.

9. Which of the following explains best thesentence in the passage "I was almost a part ofit"?

(a) I was not alone in the cell.(b) I was familiar with every bit of the cell.(c) I greeted little tufts of grass like old friends.(d) I felt quite at home in the cell.

Page 18: CSAT Introduction

CSAT

CHRO

NICLE

IAS A

CADEM

Y

18

10. The passage attempts to describe:

(a) the general conditions of the country's jails.(b) the prisoner's capacity to notice the minute

details of his surroundings.(c) the prisoner's conscious efforts to overcome

the loneliness.(d) the prisoner's ability to live happily with

other creatures.

11. The author of the passage seems to suggest that:

(a) it is possible to adjust oneself touncongenial surroundings.

(b) the conditions in Indian prisons are not bad.(c) it is not difficult to spend one's time in a

prison.(d) there is a need to improve the conditions in

our jails.

English Passage – 5

We started pitching the highest camp that has everbeen made. Everything took five times as long as itwould have taken in a place where there was enoughair to breathe; but at last we got the tent up, and whenwe crawled in, it was not too bad. There was only alight wind, and inside it was not too cold for us to takeoff our gloves. At night most climbers take off theirboots; but I prefer to keep them on. Hillary, on theother hand, took his off and laid them next to hissleeping bag.

12. What does the expression "pitching the highestcamp" imply?

(a) They reached the summit of the highestmountain in the world.

(b) Those who climbed that far earlier did notpitch any camp.

(c) So far nobody has ever climbed that high.(d) They were too many climbers and needed

to pitch a big camp.

13. They took a long time to finish the work because:

(a) they were very tired.(b) there was not enough air to breathe.(c) it was very cold.(d) it was very dark.

14. When they crawled into the tent?

(a) they took off their gloves because it wasnot very cold.

(b) they could not take off their gloves becauseit was very cold.

(c) they took off their gloves though it wasvery cold.

(d) they did not take off their gloves though itwas not cold.

English Passage – 6

A local man, staying on the top floor of an oldwooden house, was awakened at midnight by a fire.Losing his way in a smoke-filled passage, he missedthe stairway and went into another room. He pickedup a bundle to protect his face from the fire andimmediately fell through the floor below where hemanaged to escape through a clear doorway. The"bundle" proved to be the baby of the Mayor's wife.The "hero" was congratulated by all.

15. The man went into another room because:

(a) he did not know where exactly the stairwaywas.

(b) the passage was full of smoke.(c) he was extremely nervous.(d) he stumbled on a bundle.

16. The man was called a hero because he

(a) expressed his willingness to risk his life tosave others.

(b) managed to escape from the fire.(c) showed great courage in fighting the fire.(d) saved a life.

English Passage-7

He walked several miles that day but could not getanything to eat or drink except some dry bread andsome water, which he got from cottagers and farmers.As night fell, he slept under a haystack lying in ameadow. He felt frightened at first, for the wind blewawfully over the empty fields. He felt cold and hungry,and was feeling more lonely than he had ever feltbefore. He however, soon fell asleep, being much tiredwith his long walk. When he got up next day, he wasfeeling terribly hungry so he purchased a loaf of breadwith a few coins that he had.

17. When the night fell, he slept

(a) in the open field(b) under a pile of dry grass(c) in a farmer's cottage(d) under a tree

Page 19: CSAT Introduction

19CSAT

CHRO

NICLE

IAS A

CADEM

Y

18. He soon fell asleep because

(a) he was exhausted(b) he was all alone(c) he had not slept for days(d) he was very frightened

19. With reference to the passage, consider thefollowing statements :

1. He was walking through the countryside,2. The cottagers and farmers gave his enough

food so that he could sleep at night withoutfeeling hungry.

Which of the statements given above is/arecorrect?

(a) 1 only(b) 2 only(c) Both 1 and 2(d) Neither 1 nor 2

English Passage – 8

I opened the bag and packed the boots in; and then,just as I was going to close it, a horrible idea occurredto meet Had I packed my toothbrush ? I don't knowhow it is, but I never do know whether I've packedmy toothbrush. My toothbrush is a thing that hauntsme when I'm travelling, an makes my life a misery, Idream that haven't packed it, and wake up in a coldperspiration, and get out of bed and hunt for it. And,in the morning, I pack it before I have used it, and it isalways the last thing I turn out of the bag; and thenrepack and forget it, and have to rug upstairs for it atthe last moment an carry it to the railway station,wrapped u in my pocket-handkerchief.

20. When he was going to close the bag, the ideathat occurred to him was

(a) unpleasant(b) sad(c) fantastic(d) amusing

21. What makes his life miserable whenever heundertakes travelling?

(a) Going to railway station(b) Forgetting the toothbrush

(c) Packing his bag(d) Bad dreams

22. His toothbrush is finally

(a) in his bag(b) in his bed(c) in his handkerchief(d) lost

English Passage-9

In spring, polar bear mothers emerge from denswith three month old cubs. The mother bear has fastedfor as long as eight months but that does not stop theyoung from demanding full access to her remainingreserves. If there are triplets, the most persistent standsto gain an extra meal and it may have the meal at theexpense of others. The smallest of the litter forfeitsmany meals to stronger siblings. Females are protectiveof their cubs but tend to ignore family rivalry over food.In 21 years of photographing polar bears, I've onlyonce seen the smallest of triplets survive till autumn.

23. Female polar bears give birth during

(a) spnng (b) summer(c) autumn (d) winter

24. Mother bear

(a) takes sides over cubs(b) lets the cubs fend for themselves(c) feeds only their favourites(d) see that all cubs get an equal share

25. With reference to the passage, the followingassumptions have been made:

1. Polar bears fast as long as eight monthsdue to non-availability prey.

2. Polar bears always give birth to triplets.

Which of the assumptions given above is/arevalid?

(a) 1 only(b) 2 only(c) Both 1 and 2(d) Neither 1 nor 2

���

Page 20: CSAT Introduction

CSAT

CHRO

NICLE

IAS A

CADEM

Y

20

GENERAL MENTAL ABILITY

AND DATA INTERPRETATION

In the current scenario, one needs to master theart of General Mental Ability for cracking any goodcompetition. We get good number of questions onMental Ability in exams of Public Sector, Bank PO, bigB-School entrance and now in IAS as well.

For mastering General Mental Ability, firstly weshould know that what are the types of questions andwhat are the basics of the same. In mental ability, theprime thing which is being tested is your familiaritywith the numbers. The commonly asked questions arethe basics of Number Theory, basic Arithmetic, Time& Distance, Ratio & Proportion, Permutation andCombination and Probability. Now, if we look at thesetopics, all of us have read and practiced these topicsin good length at high school level. But because of ourhabit and obligations of using calculator after that hasforced us to forget all those basic concepts of arithmeticand has deviated us from the basics of mathematics.

Regarding how to improve this, there is a very basicthing that we were taught by our parents was "practicemakes a man perfect". This is very true about theMental Ability at this stage. We just need to practicethese basic concepts religiously to master theseconcepts. And believe me, there is no other way tosuccess.

To sum up all, if we want to ensure a good score inmental ability and a better second paper of IAS thisyear then probably, we need to start practicing todayand we need to clear all our basics and concepts.NCERT class sixth to tenth is the best books suited forthese preparations.

Last, but not the least practice as many samplepapers as you can along with a proper feedback anddoubt clearing of each and every paper.

Some Previous Year Questions are as follows:

1. In a rare coin collection, there is one gold coinfor every three non-gold coins. 10 more goldcoins are added to the collection and the ratioof gold coins to non-gold coins would be 1: 2.Based on the information; the total number ofcorns in the collection now becomes

(a) 90 (b) 80(c) 60 (d) 50

2. A gardener has 1000 plants: He wants to plantthem in such a way that the number of rows

and the number of columns remains the same.What is the minimum number of plants that heneeds more for this purpose?

(a) 14 (b) 24(c) 32 (d) 34

3. A sum of RS. 700 has to b used to give sevencash prizes to the students of a school for theiroverall academic performance. If each prize isRS. 20 less than its preceding prize, what is theleast value of the prize?

(a) RS. 30 (b) RS. 40(c) RS. 60 (d) RS. 80

4. Out of 120 applications for a post, 70 are maleand 80 have a driver's license. What is the ratiobetween the minimum to maximum number ofmales having driver's license?

(a) 1 to 2 (b) 2 to 3(c) 3 to 7 (d) 5 to 7

5. In a garrison, there was food for 1000 soldiersfor one month. After 10 days, 1000 more soldiersjoined the garrison. How long would the soldiersbe able to carry on with the remaining food?

(a) 25 days (b) 20 days(c) 15 days (d) 10 days

6. The tank-full petrol in Arun's motor-cycle lastsfor 10 days. If he starts using 25% moreeveryday, how many days will the tank-fullpetrol last?

(a) 5 (b) 6(c) 7 (d) 8

7. A person can walk a certain distance and driveback in six hours. He can also walk both waysin 10 hours. How much time will he take todrive both ways?

(a) Two hours(b) Two and a half hours(c) Five and a half hours(d) Four hours

8. In a class of 45 students, a boy is ranked 20th.When two boys joined, his rank was droppedby one. What is his new rank from the end?

(a) 25th (b) 26th(c) 27th (d) 28th

Page 21: CSAT Introduction

21CSAT

CHRO

NICLE

IAS A

CADEM

Y

9. A thief running at 8 km/hr is chased by apoliceman whose speed is 10 km/hr. If the thiefis 100 m ahead of the policeman, then the timerequired for the policeman to catch the thiefwill be

(a) 2 min (b) 3 min(c) 4 min (d) 6 min

10. A train travels at a certain average speed for adistance of 63 km and then travels a distanceof 72 km at an average speed of 6 km/hr morethan its original speed. If it takes 3 hours tocomplete the total journey, what is the originalspeed of the train in km/hr?

(a) 24 (b) 33(c) 42 (d) 66

11. A contract on construction job specifies apenalty for delay in completion of the workbeyond a certain date IS as follows : Rs. 200 forthe first day, Rs. 250 for the second day, Rs.300 for the third day etc., the penalty for eachsucceeding day being 50 more than that of thepreceding day. How much penalty should thecontractor pay if he delays the work by 10 days?

(a) Rs. 4950 (b) Rs. 4250(c) Rs. 3600 (d) Rs. 650

12. A student on her first 3 tests receive 4 anaverage score of N points. If she exceeds herprevious average score b. 20 points on her fourthtest, then what is the average score for the first4 tests?'

(a) N + 20 (b) N + 10(c) N + 4 (d) N + 5 “

13. In a group of persons, 70% of the persons aremale and 30% of the persons are married. Iftwo sevenths of males are married, whatfraction of the females is single?

(a) 2/7 (b) 1/3(c) 3/7 (d) 2/3

14. Consider the following figure and answer theitem that follows:

A square is divided into four rectangles as shownabove. The lengths of the sides of rectangles arenatural numbers. The areas of two rectanglesare indicated in the figure. What is the lengthof each side of the square?

(a) 10(b) 11(c) 15(d) Cannot be determined as the given data

are Insufficient

15. A person has only Rs. 1 and Rs. 2 coins withher. If the total number of coins that she has is50 and the amount of money with her is Rs. 75,then the number of Rs. 1 and Rs. 2 coins are,respectively

(a) 15 and 35 (b) 35 and 15(e) 30 and 20 (d) 25 and 25

16. Three persons start walking together and theirsteps measure 40 cm, 42 cm and 45 cmrespectively. What is the minimum distance eachshould walk so that each can cover the samedistance in complete steps?

(a) 25 m 20 cm (b) 50 m 40 cm(c) 75 m 60 cm (d) 100 m 80 cm

17. If a bus travels 160 km in 4 hours and a traintravels 320 km in 5 hours at uniform speeds,then what is the ratio of the distances travelledby them in one hour?

(a) 8 : 5 (b) 5 : 8(c) 4 : 5 (d) 1 : 2

18. There are 100 students in a particular class.60% students play cricket, 30% student playfootball and 10% student play both the games.What is the number of students who playneither cricket nor football?

(a) 25 (b) 20(c) 18 (d) 15

19. A village having a population of 4000 requires150 liters of water per head per day. It has atank measuring 20 m x 15 m x 6 m. The waterof this tank will last for

(a) 2 days (b) 3 days(c) 4 days (d) 5 days

20. Two glasses of equal volume are respectivelyhalf and three-fourths filled with milk. Theyare then filled to the brim by adding water.They are then filled to the brim by adding water.Their contents are then poured into anothervessel. What will be the ratio of milk to waterin this vessel?

(a) 1 : 3 (b) 2 : 3(c) 3 : 2 (d) 5 : 3

21. Consider the following information regardingthe performance of a class of 1000 students infour different tests:

Page 22: CSAT Introduction

CSAT

CHRO

NICLE

IAS A

CADEM

Y

22

Test I II III IV

Average marks 60 60 70 80

Range of marks 30 to 90 45 to 75 20 to 100 0 to 100

If a student scores 74 marks in each of the fourtests, in which one of the following tests is herperformance the best comparatively?

(a) Test I(b) Test II(c) Test III(d) Test IV

Data Interpretation:

In these questions data is presented either in theform of a table or a bar chart or a pie chart or a linegraph or as a combination of one of these formats.Following each of these data presentations, there willbe 4 to 6 questions. You are expected to answer thequestions by interpreting the data given in the table orgraph.

The Data Interpretation section of C-SAT isprobably closest in resemblance to the kind of problemsone will be dealing in real life situations. It tests one'sdecision-making ability and speed using limited input.Start off with topical tests in the initial stage ofpreparation.

This is the calculation intensive portion of thesection. It consists of a myriad of graphs, charts andtables from which you will have to glean and analysedata. The key to cracking this area is to quickly identifythe key pieces of data that you will require to work onthe questions asked. Sometimes questions are formedto try and bewilder students with a large amount ofdata, most of it unnecessary.

Another interesting feature of DI that you as astudent can use to your advantage is that, usually, notall questions in a set are of equal difficulty. Specifically,most sets have a 'counting' type of question (Howmany companies have profits more than x%, howmany people have incomes less than Rs. Y etc.). Mostof these questions can be solved without calculationbut by close inspection of the data presented.

Previous YearsQquestions are as follows:

Directions for the following 2 (two) items: Thefollowing pie charts show the break-up of diseasecategories recorded in the patients from two towns,Town A and Town B. Pie charts plot the diseaseCategories as percentage of the total number ofpatients. Based on these, answer the two items thatfollow the charts.

22. Which of the two towns has a higher numberof persons with Diabetes?

(a) Town A(b) Town B(c) Same in Town A and Town B(d) No inference can be drawn

23. What can we say about persons with more thanone disease from these graphs?

(a) There are likely to be persons with morethan one disease in Town A.

(b) There are likely to be persons with morethan one disease in Town B.

(c) There are likely to be persons with morethan one disease in both Towns A and B.

(d) No inference can be drawn.

Direction for the following 5 (five) items: Study thetwo figures given below and answer the five items thatfollow:

Page 23: CSAT Introduction

23CSAT

CHRO

NICLE

IAS A

CADEM

Y

24. How many Physics professors belong to the agegroup 35 - 44?

(a) 18 (b) 16(c) 14 (d) 12

25. Which one of the following disciplines has thehighest ratio of males to females?

(a) Physics (b) Mathematics(c) Chemistry (d) Economics

26. What percentage of all Psychology professorsare females?

(a) 40% (b) 50%(c) 60% (d) 70%

27. If the number of female Physics professors inthe age group 25 - 34 equals 25% of all thePhysics professors in that age group, then whatis the number of male Physics professors in theage group 25 - 34?

(a) 9 (b) 6(c) 3 (d) 2

28. If the Psychology professors in the Universityconstitute 2% of all the professors in theUniversity, then what is the number ofprofessors in the University?

(a) 400 (b) 500(c) 600 (d) 700

���

LOGICAL REASONING

& ANALYTICAL ABILITY

Logical Reasoning checks candidates’ ability toanalyze the logical foundation of a given argument.Whereas, Analytical ability is the ability to visualize,articulate & solve problems & make decisions that arebased on some available information.

Regular practice of questions can help the studentin analyzing & depicting the data in the figure. MostlyStudents think that these questions has to be solvedon the spot based on situations but practice is must toscore a good marks in these types of questions. Oncethe information has been depicted questions relatedto it can be solved easily.

Some Previous Years Questions are as follows:

1. Consider the following statements:

1. All artists are whimsical.

2. Some artists are drug addicts.3. Frustrated people are prone to become drug

addicts.

From the above three statements it may beconcluded that

(a) Artists are frustrated.(b) Some drug addicts are whimsical.(c) All frustrated people are drug addicts.(d) Whimsical people are generally frustrated.

2. Examine the following statements:

1. Either A and B are of the same age or A isolder than B.

2. Either C and D are of the same age or D isolder than C.

3. B is older than C.

Page 24: CSAT Introduction

CSAT

CHRO

NICLE

IAS A

CADEM

Y

24

Which one of the following conclusions can bedrawn from the above statements?

(a) A is older than B(b) B and D are of the same age(c) D is older than C(d) A is older than C

3. Examine the following statements:

1. Only those who have a pair of binocularscan become the members of thebirdwatcher’s club.

2. Some members of the birdwatcher’s clubhave cameras.

3. Those members who have cameras can takepart in photo-contests.

Which of the following conclusions can bedrawn from the above statements?

(a) All those who have a pair of binoculars aremembers of the birdwatcher’s club.

(b) All members of the birdwatcher’s club havea pair of binoculars.

(c) All those who take part in photo-contestsare members of the birdwatcher’s club.

(d) No conclusion can be drawn.

4. During the last summer vacation, Ankit wentto a summer camp where he took part in hiking,swimming and boating. This summer, he islooking forward to a music camp where hehopes to sing, dance and learn to play the guitar.

Based on the above information, fourconclusions, as given below, have been made.Which one of these logically follows from theinformation given above?

(a) Ankit’s parents want him to play theguitar.

(b) Ankit prefers music to outdoor activities.(c) Ankit goes to some type of camp every

summer.(d) Ankit likes to sing and dance.

5. Three persons A, B and C wore shirts of black,blue and orange colours (not necessarily in thatorder) and pants of green, yellow and orangecolours (not necessarily in that order). Noperson wore shirt and pants of the same colour.Further, it is given that

1. A did not wear shirt of black colour.2. B did not wear shirt of blue colour.3. C did not wear shirt of orange colour.4. A did not wear pants of green colour.5. B wore pants of orange colour.

What were the colours of the pants and shirtworn by C, respectively?

(a) Orange and black(b) Green and blue(c) Yellow and blue(d) Yellow and black

6. Ten new TV shows started in January—5sitcoms, 3 drama and 2 news magazines. ByApril, only seven of the new shows were stillon, five of them being sitcoms.

Based on the above information, fourconclusions, as given below, have been made.Which one of these logically follows from theinformation given above?

(a) Only one news magazine show is still on.(b) Only one of the drama shows is still on.(c) At least one discontinued shows was a

drama.

Study the following figure:

7. A person goes from A to B always moving tothe right or downwards along the Jines. Howmany different routes can he adopt?

Select the correct answer from the codes givenbelow:

(a) 4 (b) 5(c) 6 (d) 7

8. Read the passage given below and the twostatements that follows (given on the basis ofthe passage) :

Four men are waiting at Delhi airport for a Mumbaiflight. Two are doctors and the other two arebusinessmen. Two speak Gujarati and two speakTamil. No two of the same profession speak the samelanguage. Two are Muslims and two are Christians.No two of the same religion are of the same profession,nor do they speak the same language. The Tamil-speaking doctor is a Christian.

1. The Christian-businessman speaks Gujarati.2. The Gujarati-speaking doctor is a Muslim.

Page 25: CSAT Introduction

25CSAT

CHRO

NICLE

IAS A

CADEM

Y

What of the above statements is/are correctconclusion/conclusions?

(a) 1 only (b) 2 only(c) Both 1 and 2 (d) Neither 1 nor 2

9. Consider the following statement:

“Though quite expensive, television is not aluxury item, as one can learn many thingsthrough television.”

Which one of the following is a valid inferencefrom the above statement?

(a) All expensive things is a valid inference fromthe above statement.

(b) All essential things for learning are notluxury.

(c) Television is essential for learning.(d) Television is not a luxury item.

10. Gita is prettier than Sita but not as pretty asRita. Then,

(a) Sita is not as pretty as Gita.(b) Sita is prettier than Rita.(c) Rita is not as pretty as Gita.(d) Gita is prettier than Rita.

Given that,

1. A is the brother of B.2. C is the father of A.3. D is the brother of E.4. E is the daughter of B.

Then, the uncle of D is

(a) A (b) B(c) C (d) E

11. Examine the following statements:

1. Rama scored less than Ratna.2. Rani scored less than Ratna.3. Ratna scored more than Rama.4. Padma scored more than Rama but less

than Ratna.

Who scored the highest?

(a) Rama (b) Padma(c) Rani (d) Ratana

Directions for the following 5 (five) items: Examinethe information given in the following paragraph andanswer the items that follow:

Guest lectures on five subjects viz., Economics,History, Statistics, English and Mathematics have tobe arranged in a week from Monday to Friday. Onlyone lecture can be arranged on each day. Economicscannot be scheduled on Tuesday. Guest faculty for

History is available only on Tuesday. Mathematicslecture has to be scheduled immediately after the dayof Economics lecture. English lecture has to bescheduled immediately before the day of Economicslecture.

12. Which lecture is scheduled on Monday?

(a) History (b) Economics(c) Mathematics (d) Statistics

13. Which lecture is scheduled between Statisticsand English?

(a) Economics (b) History(c) Mathematics (d) No lecture

14. Which lecture is the last one in the week?

(a) History (b) English(c) Mathematics (d) Economics

15. Which lecture is scheduled on Wednesday?

(a) Statistics (b) Economics(c) English (d) History

16. Which lecture is scheduled before theMathematics lecture?

(a) Economics (b) History(c) Statistics (d) English

17. Consider the following statements:

1. All machines consume energy.2. Electricity provides energy.3. Electrically operated machines are cheap

to maintain.4. Electrically operated machines do not cause

pollution.

Which one of the following inferences can bedrawn from the above statements?

(a) All machines are run by electric energy.(b) There is no form of energy other than

electricity.(c) Most machines are operated on electric

energy.(d) Electrically operated machines are

preferable to use.

18. Examine the following statements:

1. None but the rich can afford air-travel.2. Some of those who travel by air become

sick.3. Some of those who become sick require

treatment.

Which one of the following conclusions can bedrawn from the above statements?

(a) All the rick persons travel by air.

Page 26: CSAT Introduction

CSAT

CHRO

NICLE

IAS A

CADEM

Y

26

(b) Those who travel by air become sick.(c) All the rick persons become sick.(d) All those who travel by air are rich.

19. In five flats, one above the other, live fiveprofessionals. The professor has to go up to meethis IAS officer friend. The doctor is equalfriendly to all, and has to go up as frequentlyas go down. The engineer has to go up to meethis MLA friend above whose flat lives theprofessor's friend.

From the ground floor to the top floor, in whatorder do the five professionals live?

(a) Engineer, Professor, Doctor, IAS officer,MLA

(b) Professor, Engineer, Doctor, IAS officer,MLA

(c) IAS officer, Engineer, Doctor, Professor,MLA

(d) Professor, Engineer, Doctor, MLA, IASofficer

20. Only six roads A, B, C, P, Q, and R connect amilitary camp to the rest of the country. Onlyone out of A, P and R is open at any one time.If B is open during storms. P is closed duringfloods. In this context, which one of thefollowing statements is correct?

(a) Under normal conditions only three roadsare open.

(b) During storms at least one road is open.(c) During floods only three roads are open.(d) During calamities all roads are closed.

21. Four political parties W, X Y and Z decided toset up a joint candidate for the comingparliamentary elections. The formula agreed bythem was the acceptance of to candidate bymoist of the parties. Four aspiring candidates,A, B, C and D approached the parties for theirtickets.

A was acceptable to W but not to Z.

B was acceptable to Y but not to X.

C was acceptable to W and Y.

D was acceptable to W and X.

When candidate B was preferred by W and Z,candidate C was preferred by X and Z , andcandidate A was acceptable to X but not to Y,who got the ticket?

(a) A (b) B(c) C (d) D

22. Six squares are coloured, front and back, red(R), blue (B), yellow (Y), green (G), white (W)and orange (O) and are hinged together asshown in the figure given below. If they arefolded to form a cube, what would be the faceopposite the white face?

(a) R (b) G(c) B (d) O

23. In the above figure, circle P representshardworking people, circle Q representsintelligent people, circle R represents truthfulpeople, and circle S represents honest people.Which region represents the people who areintelligent, honest and truthful but nothardworking?

(a) 6 (b) 7(c) 8 (d) 11

24. Consider the following figures 1, 2, 3 and 4:

In the figure from 1 to 4 above, two symbolsare shown to change their position in a regulardirection. Following the same sequence, whichone of the following will appear at the fifthstage?

Directions for the following 2 (two) items: In eachitem, there are two sets of figures; first four figuresnamed Problem figures and next four figures namedAnswer figures indicated as (a), (b), (c) and (d). Theproblem figures follow a particular sequence. In

Page 27: CSAT Introduction

27CSAT

CHRO

NICLE

IAS A

CADEM

Y

accordance with the same, which one of the fouranswer figures should appear as the fifth figure?

25. Problem figures:

Answer figures:

26. Problem figures:

Answer figures:

27. Consider the following diagrams:

x men, working at constant speed, do a certainjob in y days. Which one of these diagramsshows the relation between x and y?

(a) diagram I (b) diagram II(c) diagram III (d) diagram IV

28. Consider the following matrix:

3 370 7

2 224 6

1 730 X

What is the number at 'X' in the above matrix?

(a) 5 (b) 8(c) 9 (d) 11

29. Four cars are hired at the rate of Rs. 6 per kmplus the cost of diesel at Rs. 40 a litre. In thiscontext, consider the details given in thefollowing table:

Car Mileage Hours Total(km/l) Payment (Rs.)

A 8 20 2120

B 10 25 1950

C 9 24 2064

D 11 22 1812

Which car maintained the maximum averagespeed?

(a) Car A (b) Car B(c) Car C (d) Car D

30. Examine the following three figures in whichthe numbers follow a specific pattern:

The missing number (?) in the third figure aboveis

(a) 7 (b) 16(c) 21 (d) 28

31. A cube has six numbers marked 1, 2, 3, 4, 5and 6 on its faces. Three views of the cube areshown below:

(a) 2 and 3 (b) 6 and 1(c) 1 and 4 (d) 3 and 1

Page 28: CSAT Introduction

CSAT

CHRO

NICLE

IAS A

CADEM

Y

28

32. Consider the following figures:

Which one of the following figures wouldlogically come in the 7th position indicatedabove by a question mark?

33. Four friends, A, B, C and D distribute somemoney among themselves in such a manner thatA gets one less than B, C gets 5 more than D,D gets 3 more than B. Who gets the smallestamount?

(a) A (b) B(c) C (d) D

Directions for the following 4 (four) items: Read thefollowing statements and answer the four items thatfollow:

Five cities P, Q, R, S and T are connected by differentmodes of transport as follows:

P and Q are connected by boat as well as rail.

Sand R are connected by bus and boat.

Q and T are connected by air only.

P and R are connected by boat only.

T and R are connected by rail and bus.

34. Which mode of transport would help one toreach R starting from Q, but without changingthe mode of transport?

(a) Boat (b) Rail(c) Bus (d) Air

35. If a person visits each of the places startingfrom P and gets back to P, which of thefollowing places must he visit twice?

(a) Q (b) R(c) S (d) T

36. Which one of the following pairs of cities isconnected by any of the routes directly withoutgoing to any other city?

(a) P and T (b) T and S(c) Q and R (d) None of these

37. Between which two cities among the pairs ofcities given below are there maximum traveloptions available?

(a) Q and S (b) P and R(c) P and T (d) Q and R

Directions for the following 3 (three) items: Read thefollowing passage and answer the three items thatfollow:

A tennis coach is trying to put together a team offour players for the forthcoming tournament. For this7 players are available: males A, Band C; and femalesW, X, Y and Z. All players have equal capability andat least 2 males will be there in the team. For a team offour, all players must be able to play with each other.But, B cannot play with W, C cannot play with Z andW cannot play with Y.

38. If Y is selected and B is rejected, the team willconsist of which one of the following groups?

(a) A, C, Wand Y(b) A, C, X and Y(c) A, C, Y and Z(d) A, W, Y and Z

39. If B is selected and Y is rejected, the team willconsist of which one of the following groups?

(a) A, B, C and W (b) A, B, C and Z(c) A, B, C and X (d) A, W, Y and Z

40. If all the three males' are selected, then howmany combinations of four member teams arepossible?

(a) 1 (b) 2(c) 3 (d) 4

41. The music director of a film wants to selectfour persons to work on "different aspects ofthe composition of a piece of music. Sevenpersons are available for this work; they areRohit, Tanya, Shobha, Kaushal, Kunal, Mukeshand J aswant. Rohit and Tanya will not worktogether. Kunal and Shobha will not worktogether. Mukesh and Kunal want to work

Page 29: CSAT Introduction

29CSAT

CHRO

NICLE

IAS A

CADEM

Y

together. “Which of the following is the mostacceptable group .of people that can be selectedby the music director?

(a) Rohit, Shobha, Kunal and Kaushal(b) Tanya, Kaushal, Shobha and Rohit(c) Tanya, Mukesh, Kunal and Jaswant(d) Shobha, Tanya, Rohit and Mukesh

42. Five people A, B, C, D and E are, seated abouta round table, Every chair is spaced equidistantfrom adjacent chairs,

I. C is seated next to A.II. A is seated two seats from D.

III. B is not seated next to A.

Which of the following must be true?

I. D is seated next to B.II. E is seated next to A.

Select the correct answer from the codes givenbelow:

(a) I only (b) II only(c) Both I and II (d) Neither I nor II

Directions for the following 3 (three) items: Examinecarefully the following statements and answer the threeitems that follow:

Out of four friends A, B, C and D, “A and B playfootball and cricket, “Band C play cricket and hockey,“A and D play basketball and football, “C and D playhockey and basketball.

43. Who does not play hockey?

(a) D (b) C(c) B (d) A

44. Who plays football, basketball and hockey?

(a) D (b) C(c) B (d) A

45. Which game do B, C and D play?

(a) Basketball (b) Hockey(c) Cricket (d) Football

46. Geeta is older than her cousin Meena, Meena'sbrother Bipin is older than Geeta. When Meenaand Bipin visit Geeta, they like to play chess.Meena wins the game more often than Geeta.Based on the above information, fourconclusions, as given below, have been made.Which one of these logically follows from theinformation given above?

(a) While playing chess with Geeta and Meena,Bipin often loses.

(b) Geeta is the oldest among the three.

(c) Geeta hates to 10 the game.(d) Meena is the youngest of the three.

47. The followings figures has four curves namelyA, B, C and D, Study the figure and answerthe item that follows.

Which curve indicates the exponential growth?

(a) A (b) B(c) C (d) D

48. Consider the four age pyramids given belownamely A, B, C and D representing four differentcountries.

Which one of them indicates the decliningpopulation?

(a) A (b) B(c) C (d) D

49. Consider the following distance - time graph.The graph shows three athletes A, Band Crunning side by side for a 30 km race.

Page 30: CSAT Introduction

CSAT

CHRO

NICLE

IAS A

CADEM

Y

30

DECISION MAKING AND

INTERPERSONAL SKILLS

With reference to the above graph consider thefollowing statements :

1. the race was won by A.2. B was ahead of A up to 25 km 26 mark.

���

Decision Making can be regarded as the mentalprocesses resulting in the relation of a course of actionamong several alternative scenarios. WhereasInterpersonal skills are the skills that a person uses tointeract with other people. It is also sometimes calledcommunication skills. Positive interpersonal skillsincrease the productivity of the organization since thenumbers of conflicts are reduced. It also allowscommunication to be easy and comfortable.

UPSC has not applied negative marking to thequestions of decision making because decision makingis based on individual logical & verbal ability and canvary from person to person. Thus attempting them isthe safest part in paper but proper understanding ofthe question is must.

Decision Making Process

(a) Objectives must be established first and placedin order of preference.

(b) Alternative actions must be developed.(c) The alternative that is able to achieve all the

objectives is the tentative decision.(d) The tentative decision is evaluated for more

possible consequences.

Decision making steps:

1st Step : Outline your goal and outcome.

2nd Step : Gather data. This will help the decisionmakers having actual evidence to help them come upwith a solution.

3rd Step : Brainstorm to develop alternatives.Coming up with more than one solution, enables youto see which one can actually work.

4th Step : List pros and cons of each alternative,with the help of which, you can eliminate the solutionsthat have more cons than pros, making your decisioneasier.

5th Step : Make the decision by picking the onethat has many pros, and the one that everyone canagree with.

6th Step : Immediately take action. Once thedecision is picked, you should implement it right away.

7th Step : Learn from and reflect on the decisionmaking. This step allows you to gauge where you wereright or wrong while implementing the decision.

Some Previous Year Questions are as follows:

Directions for the following questions:

Each item describes a situation and is followed byfour possible responses. Indicate the response you findmost appropriate. Choose only one response for eachitem. The responses will be evaluated based on the levelof appropriateness for the given situation.

Please attempt all the items. There is no penaltyfor, wrong answers for these items.

1. You are the head of your office. There arecertain houses reserved for the allotment to theoffice staff and you have been given thediscretion to do so. A set of rules for theallotment of the houses has been laid down byyou and has been made public. Your personalsecretary, who is very close to you, comes toyou and pleads that as his father is seriously ill,he should be given priority in allotment of ahouse. The office secretariat that examined therequest as per the rules turns down the requestand recommends the procedure to be followedaccording to the rules. You do not want toannoy your personal secretary. In suchcircumstances, what would you do?

(a) Call him over to your room and personallyexplain why the allotment cannot be done.

(b) Allot the house to him to win his loyalty.

3. C ran very slowly from the begining.Which of the statements given above is/are correct?

(a) 1 only (b) 1 and 2 only(c) 2 and 3 only (d) 1, 2 and 3

Page 31: CSAT Introduction

31CSAT

CHRO

NICLE

IAS A

CADEM

Y

(c) Agree with the office note to show thatyou are not biased and that you do notindulge in favouritism.

(d) Keep the file with you and not pass anyorders.

2. While travelling in a Delhi-registered commercialtaxi from Delhi to an adjacent city (anotherState), your taxi driver informs you that as hehas no permit for running the taxi in that city,he will stop at its Transport Office and pay theprescribed fee of Rs. forty for a day. Whilepaying the fee at the counter you find that thetransport clerk is taking an extra fifty rupeesfor which no receipt is being given. You are ina hurry for your meeting. In such circumstances,what would you do?

(a) Go up to the counter and ask the clerk togive back the money which he has illegallytaken.

(b) Do not interfere at all as this is a matterbetween the taxi driver and the taxauthorities.

(c) Take note of the incident and subsequentlyreport the matter to the concernedauthorities.

(d) Treat it as a normal affair and simply forgetabout it.

3. A person lives in a far off village which is almosttwo hours by bus. The villager's neighbour is avery powerful landlord who is trying to occupythe poor villager's land by force. You are theDistrict Magistrate and busy in a meeting calledby a local Minister. The villager has come allthe way, by bus and on foot, to see you andgive an application seeking protection from thepowerful landlord. The villager keeps on waitingoutside the meeting hall for an hour. You comeout of the meeting and are rushing to anothermeeting. The villager follows you to submit hisapplication. What would you do?

(a) Tell him to wait for another two hours tillyou come back from your next meeting.

(b) Tell him that the matter is actually to bedealt by a junior officer and that he shouldgive the application to him.

(c) Call one of your senior subordinate officersand ask him to solve the villager's problem.

(d) Quickly take the application from him, askhim a few relevant questions regarding hisproblem and then proceed to the meeting.

4. There IS a shortage of sugar in your Districtwhere you are the District Magistrate. TheGovernment has ordered that only a maximum

amount of 30 kg sugar is to be released forwedding celebrations. A son of your close friendis getting married and your friend requests youto release at least 50 kg sugar for his son'swedding. He expresses annoyance when youtell him about the Government's restrictions onthis matter. He feels that since you are theDistrict Magistrate you can release any amount.You do not want to spoil your friendship withhim. In such circumstances, how would youdeal with the situation?

(a) Release the extra amount of sugar whichyour friend has requested for

(b) Refuse your friend the extra amount andstrictly follow the rules.

(c) Show your friend the copy of theGovernment instructions and then persuadehim to accept the lower amount asprescribed in the rules.

(d) Advise him to directly apply to the allottingauthority and inform him that you do notinterfere in this matter.

5. You are in-charge of implementing the FamilyPlanning programme in an area where there isa strong opposition to the present policy. Youwant to convince the residents of the need forkeeping small families. What would be the bestway of communicating this message?

(a) By logically explaining to the residents theneed for family planning to improve thehealth and living standards.

(b) By encouraging late marriages and properspacing of children.

(c) By offering incentives for adopting familyplanning devices.

(d) By asking people who have been sterilizedor are using contraceptives to directly talkto the residents.

6. You are a teacher in a University and are settinga question paper on a particular subject. Oneof your colleagues, whose son is preparing forthe examination on that subject, comes to youand informs you that it is his son's last chanceto pass that examination and whether youcould help him by indicating what questionsare going to be in the examination. In the past,your colleague had helped you in anothermatter. Your colleague informs you that his sonwill suffer from depression if he fails in thisexamination. In such circumstances, whatwould you do?

(a) In view of the help he had given you,extend your help to him.

Page 32: CSAT Introduction

CSAT

CHRO

NICLE

IAS A

CADEM

Y

32

(b) Regret that you cannot be of any help tohim.

(c) Explain to your colleague that this wouldbe violating the trust of the Universityauthorities and you are not in a position tohelp him.

(d) Report the conduct of your colleague to thehigher authorities.

7. You have differences of opinion regarding thefinal report prepared by your subordinate thatis to be submitted urgently. The subordinate isjustifying the information given in the report.You would….

(a) Convince the subordinate that he is wrong.(b) Tell him to reconsider the results.(c) Revise the report on your own.(d) Tell him not to justify the mistake.

8. You are competing with your batch-mate for aprestigious award to be decided based on anoral presentation. Ten minutes are allowed foreach presentation. You have been asked by thecommittee to finish on time. Your friend,however, is allowed more than the stipulatedtime period. You would….

(a) Lodge a complaint to the chairpersonagainst the discrimination.

(b) Not listen to any justification from thecommittee.

(c) Ask for withdrawal of your name.(d) Protest and leave the place.

9. You are handling a time-bound project. Duringthe project review meeting, you find that theproject is likely to get delayed due to lack ofcooperation of the team members. You would…

(a) Warn the team members for their non-cooperation.

(b) Look into reasons for non-cooperation.(c) Ask for the replacement of team members.(d) Ask for extension of time citing reasons.

10. You are the chairperson of a state sportscommittee. You have received a complaint andlater it was found that an athelete in the junioirage category who has won a medal has crossedthe age criteria by 5 days. You would….

(a) Ask the screening committee for aclarification.

(b) Ask the athlete to return the medal.(c) Ask the athlete to get an affidavit from the

court declaring his/her age.(d) Ask the members of the sports committee

for their views.

11. You are handling a priority project and havebeen meeting all the deadlines and are thereforeplanning your leave during the project. Yourimmediate boss does not grant your leave citingthe urgency of the project. You would….

(a) Proceed on leave without waiting for thesanction.

(b) Pretend to be sick and take leave.(c) Approach higher authority to reconsider the

leave application.(d) Tell the boss that it is not justified.

12. You are involved in setting up a water supplyproject in a remote area. Full recovery of cost isimpossible in any case. The income levels in thearea are low and 25% of the population is belowpoverty line (BPL). When a decision has to betaken on pricing you would….

(a) Recommend that the supply of water befree of charge in all respects.

(b) Recommend that the users pay a one timefixed sum for the installation of taps andthe usage of water be free.

(c) Recommend that a fixed monthly chargebe levied only on the non-BPL families andfor BPL families water should be free.

(d) Recommend that the users pay a chargebased on the consumption of water withdifferentiated charges for non-BPL and BPLfamilies.

13. As a citizen you have some work with agovernment department. The official calls youagain and again; and without directly askingyou, sends out feelers for a bribe. You want toget your work done. You would…..

(a) Give a bribe.(b) Behave as if you have not understood the

feelers and persist with your application.(c) Go to the higher officer for help verbally

complaining about feelers.(d) Send in a formal complaint.

14. You have been asked to give an explanation fornot attending an important official meeting.Your immediate boss who has not informed youabout the meeting is now putting pressure onyou not to place an allegation against him /her. You would

(a) send a written reply explaining the fact.(b) seek an appointment with the top boss to

explain the situation.(c) admit your fault to save the situation.(d) put the responsibility on the coordinator of

the meeting for not informing.

Page 33: CSAT Introduction

33CSAT

CHRO

NICLE

IAS A

CADEM

Y

15. A local thug (bad element) has started illegalconstruction on your vacant plot. He has refusedyour request to vacate and threatened you ofdire consequences in case you do not sell theproperty at a cheap price to him. You would

(a) sell the property at a cheap price to him.(b) go to the police for necessary action.(c) ask for help from your neighbours.(d) negotiate with the goon to get a higher

price.

16. You have to accomplish a very important taskfor your headquarters within the next two days.Suddenly you meet with an accident. Youroffice insists that you complete the task. Youwould

(a) ask for an extension of deadline.(b) inform Headquarters of your inability to

finish on time.(c) Suggest alternate person to headquarters

who may do the needful.(d) stay away till you recover.

17. You are an officer-in-charge for providing basicmedical facilities to the survivors of anearthquake affected area. Despite your bestpossible effort, people put allegations againstyou for making money out of the funds givenfor relief. You would

(a) let an enquiry be set up to look into thematter.

(b) ask your senior to appoint some otherperson in your place.

(c) not pay attention to allegations.(d) stop undertaking any initiative till the

matter is resolved.

18. You have been made responsible to hire boatsat a short notice to be used for an area underflood. On seeing the price mentioned by theboat owners you found that the lowest pricewas approximately three times more than theapproved rate of the Government. You would

(a) reject the proposal and call for a fresh price.(b) accept the lowest price.(c) refer the matter to the Government and

wait.

(d) threaten the boat owners about a possiblecancellation of the licence.

19. You are the officer-in-charge of a villageadministering distribution of vaccine in anisolated epidemic hit village, and you are leftwith only one vaccine. There is a requirementof that vaccine from the Gram Pradhan andalso a poor villager. You are being pressurisedby the Gram Pradhan to issue the vaccine tohim. You would

(a) initiate the procedure to expedite the nextsupply without issuing the vaccine to either.

(b) arrange vaccine for the poor villager fromthe distributor of another area.

(c) ask both to approach a doctor and get aninput about the urgency.

(d) arrange vaccine for the Gram Pradhan fromthe distributor of another area.

20. You have taken up a project to create night-shelters for homeless people during the winterseason. Within a week of establishing theshelters, you have received complaints from theresidents of the area about the increase in theftcases with a demand to remove the shelters.You would

(a) ask them to lodge a written complaint inthe police station.

(b) assure residents of an enquiry into thematter.

(c) ask residents to consider the humanitarianeffort made.

(d) continue with the project and ignore theircomplaint.

21. You, as an administrative authority, have beenapproached, by the daughter-in-law of aninfluential person regarding harassment by herin-laws on account of insufficient dowry. Herparents are not able to approach you becauseof social pressures. You would

(a) call the in-laws for an explanation.(b) counsel the lady to adjust, given such a

circumstance.(c) take action after her parents approach you.(d) ask her to lodge complaint with the police.

���